<<

CONTRACTS MNEMONICS

1) The ingredients for a valid are TACO: T – Definite TERMS, express or implied A – of terms C – O – OFFER inviting acceptance

2) An offer expires when it gets TIRED: T – Reasonable TIME after an offer is made, or after expiration date expressly stated in an offer I – Mental INCAPACITY or death of offeror or offeree R – REVOCATION of an offer communicated to an offeree before acceptance E – EXPRESS or implied rejection communicated to offeror D – DESTRUCTION of the subject matter of the offer or intervening illegality terminates an offer by operation of law

3) Options can DIE by: D – DESTRUCTION of subject matter I – Intervening ILLEGALITY E – EXPIRATION of a stated option time extinguishes the option

4) In NY, a signed writing takes the place of consideration for POP: P – PRE–EXISTING duty O – Contract OPTIONS P – PAST consideration (which must be recited in the signed writing)

5) Generally, contracting parties are free to modify a 3rd party beneficiary (3PB) K, unless, prior to receiving notice of the K modification, the 3PB got MAD: M – MANIFESTED an assent called for in the 3PB K, at the request of one of the contracting parties (i.e., accepted a K offer arising from the 3PB K) A – Commenced a breach of K ACTION against the promisor, or D – DETRIMENTALLY relied on the K

©2015 Pieper Bar Review 1

6) Contract assignments may involve the ADA: A – of a contractual right to collect money owed under the K D – of the performance required under the K A – ASSUMPTION of liability for performing the K

7) A gratuitous assignment becomes irrevocable, and a second assignee prevails over a prior assignee of the contract when J.P.N.C.: J – Recovers a JUDGMENT P – Gets PAID NC – Enters a NEW CONTRACT

8) Absent express language in a K prohibiting assignment, K rights are freely assignable, except those of SIR P: S – Where a STATUTE expressly prohibits the assignment of a K right (but if that claim is reduced to judgment, it is assignable) I – Where the assignment is coupled with an IMPROPER delegation of a duty under the K to a person unqualified to fulfill that duty R – Where the assignment increased the RISK to the other contracting party P – Where the services to be rendered are highly PERSONAL in nature (because that would materially alter the bargain)

9) In New York, by statute (see SIR P), you cannot assign a WASP: W – WORKER’S COMPENSATION A – ALIMONY or child support payments S – SPENDTHRIFT TRUSTS P – PERSONAL INJURY or wrongful death causes of action

10) Look at HAIL to determine whether a breach is material or immaterial: H – HARDSHIP on breaching party if total material breach is declared A – AMOUNT of benefit bestowed on non–breaching party I – Whether breach was INNOCENT L – LIKELIHOOD of full performance being achieved

11) defenses are I3 FU2MED & I S2IP: I – INFANCY I – INSANITY – INCOMPETENCY I – INTOXICATION F – FRAUD U – U – M – E – EQUITABLE DEFENSES D – DURESS

I – of performance

S – S – STATUTE OF LIMITATIONS I – ILLEGALITY P – PAROLE RULE

©2015 Pieper Bar Review 2

12) Lack of contractual arises from the 3 I’s: I – INFANCY I – INTOXICATION I – Mental INFIRMITY

13) SI2R M is a fraud: S – SCIENTER I – D lied with an INTENT to defraud the P I – P suffered an economic INJURY R – P justifiably RELIED on D’s M – D misrepresented a MATERIAL fact, which induced P to enter the K

14) A unilateral mistake in calculating figures may allow the mistaken party the remedy of equity of rescission, if he calls the COPS: C – The computational mistake was COMMUNICATED to the other party before that person changed his/her position in reliance on those mistaken figures O – The mistake involved was one of ORDINARY negligence P – The mistaken party gave PROMPT notice of the mistake S – The mistake will impose SUBSTANTIAL hardship on the party if not corrected

15) The following SMART FLYS must be in writing, subscribed by the party to be charged with the breach (i.e., must contain defendant’s signature): S – SURETY contracts M – MARRIAGE contracts A – ANSWER for debts discharged in bankruptcy R – REAL contracts T – TESTAMENTARY promises (NY ONLY) F – FINDERS FEE arrangements L – longer than 1 year Y – Contracts not capable of complete performance within 1 YEAR S – UCC Article 2 SALES CONTRACTS for $500 or more

16) Use a COMB for promissory in NY: C – CHARITABLE pledges O – To avoid OUTRAGEOUSLY unconscionable results M – Oral MARRIAGE contracts B – Promises by gratuitous BAILEES to obtain insurance on bailed goods

17) There are 4 T–CUP elements for a constructive trust: T – TRANSFER of property in reliance on promise C – Existence of CONFIDENTIAL or fiduciary relationship U – to transferee of property or to some third party, AND P – PROMISE, express or implied, to hold property for plaintiff’s benefit, which promise has been breached

©2015 Pieper Bar Review 3

18) A THUG may render an illegal contract enforceable, based on: T – TYPE of illegality & extent to which the public is harmed H – HARM that forfeiture would cause if contract was declared unenforceable due to illegality; ct looks to see whether contract has been substantially performed U – UNJUST enrichment (a windfall) to party asserting illegality defense G – Relative GUILT of each party

19) The theory of impossibility frequently involves the 4 Ds: D – DEATH D – DANGER to life/ill health D – DESTRUCTION of the subject matter of the law suit D – DELAYS, temporarily causing performance to become impracticable or impossible

20) OF MICE2 permits parole evidence: O – To establish an ORAL condition precedent to legal effectiveness of contract, provided it doesn’t contradict express term(s) of the contract F – A party cannot invoke the Parole Evidence Rule to shield that party from allegations of FRAUD or Misrepresentation M – To establish MUTUAL Mistake or claim for reformation of contract I – To establish ILLEGALITY C – To establish failure of CONSIDERATION E – To EXPLAIN ambiguous or missing terms E – To show that no ENFORCEABLE agreement was ever intended

21) Contract law does not allow damages recovery for CAPS: C – To recover consequential damages, unless they were within the CONTEMPLATION OF BOTH PARTIES when the contract was executed A – Damages that party could have AVOIDED P – Damages for PAIN & suffering or emotional distress resulting from a breached contract, even if such damages were foreseeable S – SPECULATIVE damages aren’t recoverable (all damages must be proven within a reasonable certainty)

22) Generally, parties can put whatever terms they’d like into a K, except for PLUS: P – Terms that violate PUBLIC POLICY L – Terms providing for an excessive amount of U – Terms that are UNCONSCIONABLE S – Clauses providing that one party can seek in the event of a breach (the contract does NOT have to enforce these clauses)

23) Apply a TISSUE to a covenant restricting a former employee from competing: T – TIME restriction must be reasonable (usually two years or fewer) I – INABILITY of the employee to gain work elsewhere S – The geographic SPACE/SCOPE of the restriction must be as narrow as possible (must only be to the extent necessary to protect employer’s interest) SUE – The employee services must be SPECIAL, UNIQUE, or EXTRAORDINARY

©2015 Pieper Bar Review 4

SALES MNEMONICS

24) ICOP limits the Perfect Tender Rule: I – INSTALLMENT contracts C – Timely delivery was COMMERCIALLY IMPRACTIBLE by an event not contemplated by the parties. O – Delivery in , OBJECTIVELY and reasonably believing the goods would be acceptable to the buyer P – PRIOR TO DELIVERY DATE set forth in the contract, conforming goods are delivered to replace the nonconforming goods

25) Additional terms will not be added to the contract when OCAN: O – The offeror OBJECTS to additional terms within a reasonable time C – The offer expressly CONDITIONS the agreement on accepting the terms in the offer as they are A – The additional terms materially ALTER the offer N – Either or both parties are NON–MERCHANTS

26) A J STRAW clause materially alters an offer if it would cause surprise or hardship to the offeror if the offeror was not made aware of its existence: J – Bestowing JURISDICTION on a particular court, or requiring offeror to consent to jurisdiction in particular state S – Shortening the STATUTE OF LIMITATIONS to sue for non–conforming goods T – Limiting liability or limiting a buyer’s right to sue for consequential damages R – Altering UCC rules for A – Adding an (unless customary to do so in the trade) W – Adding a clause negating a WARRANTY (e.g., one of merchantability or fitness)

27) Exceptions to the Statute of Frauds requirement are SWAMP: S – Contracts for SPECIALLY manufactured goods W – WAIVER A – Judicial ADMISSION of contract M – “MERCHANT MEMORANDUM” P – PART PERFORMANCE

28) If a sales contract is silent on a topic, the UCC implies the following CIDER rules: C – Seller is not obligated to extend CREDIT to the buyer I – Buyer has the right to INSPECT the seller’s tendered goods (except no right to inspect when the transaction involves a bill of lading) D – Seller’s tender of DELIVERY is implied to be at seller’s place of business, unless both parties know that the goods are located elsewhere E – Buyer and seller must EXCHANGE performance concurrently R – RISK OF LOSS is on the party in the best position to bear that risk

©2015 Pieper Bar Review 5

29) SOAL–V and SORE–V affect risk of loss: SOAL V – SALE ON APPROVAL LATE VESTING (goods held by the buyer are not subject to claims of the buyer’s creditors) SORE V – SALE OR RETURN EARLY VESTING (title and ROL vest immediately in the buyer, even though the buyer has a right to rescind the K)

30) Remedies available to a seller are SPARKLE: S – STOPPING goods in transit P – Suing for the entire contract PRICE A – Demanding ASSURANCES R – RE–SELLING goods to another buyer K – KEEPING part of a breaching buyer’s deposit, never more than $500 L – Suing for LOST Profit E – EXERCISING the right to reclaim goods delivered to the insolvent buyer

31) Remedies available to a buyer are CID’S WAR: C – I – INCIDENTAL & consequential damages D – DAMAGES for lost benefit of the bargain, or for the price paid S – SPECIFIC PERFORMANCE on a contract for unique goods W – Breach of WARRANTY A – ACCEPTANCE revocation R – REJECTING non–conforming goods

32) A sales contract contains M FEET warranties: M – Warranty of MERCHANTABILITY F – Warranty of FITNESS for a particular purpose E – Warranty against ENCUMBRANCES E – EXPRESS warranties T – Warranty of TITLE

33) Express warranties are SAD: S – SAMPLE or model, which is the basis of the bargain A – Written or oral AFFIRMATION of fact or promise made by the seller relating to the goods D – DESCRIPTION of the goods in advertisements, brochures, or catalogs

34) A P’s claim against a seller for a defective good can be based on one or more overlapping but different PINE theories of liability: P – theory of strict PRODUCTS liability I – Contract theory for breach of N – Torts theory of NEGLIGENCE E – Contract theory for breach of an EXPRESS warranty

©2015 Pieper Bar Review 6

35) G.P.S. LAMP can use the following defenses against a breach of warranty claim: G – GOVERNMENT military contract defense P – Federal PREEMPTION S – STATUTE OF LIMITATIONS L – LACK of timely notice to a seller A – ASSUMPTION of risk (can be asserted against any PINE claim) M – Unforeseen MISUSE of a product P – Lack of PRIVITY of contract

36) Strict products liability is imposed on a regular seller of a DUD product: D – DEFECTIVE, and an U – UNREASONABLY D – DANGEROUS product

37) When asserting a strict products liability claim, P must prove that a DIM dangerous defect in the product proximately caused a physical injury: D – DESIGN defect I – INADEQUATE warning M – Mistake in the MANUFACTURING process

©2015 Pieper Bar Review 7

Damages The Goal: Contract law seeks to provide the non–breaching party with its , which place the non–breaching party in as good a position as if the breaching party had performed its obligations under the contract. Expectation Damages are the usual form of damages awarded when a contract is breached.

The General Formula: The Value of the Promised Performance MINUS The Value of what the Plaintiff Received PLUS Any Incidental and/or Consequential Damages MINUS Any Costs that Were/Could have been Avoided/Mitigated

To make use of this formula, it helps to know something about each of its component parts:

The “value of the promised performance” is typically just the contract price, though a fact pattern could tell you that the value of the performance was something else, for example, if the buyer was getting a good deal/discounted price. By allowing a plaintiff to recover the value of the promised performance as opposed to just the contract price, the plaintiff’s “benefit of the bargain” is preserved.

The “value of what the plaintiff received” is exactly what it says: some value associated with the defendant’s performance (this value will be identified in the fact pattern, if necessary).

“Incidental damages” are any “costs incurred in a reasonable effort, whether successful or not, to avoid loss, as where a party pays brokerage fees in arranging or attempting to arrange a substitute transaction.” Restatement 2d Contracts § 347, comment c. In a sale of goods contract, incidental damages are awarded to a non–breaching buyer for any reasonable expenses arising out of the breach including those incurred through cover, or any cost “reasonably incurred in inspection, receipt, transportation and care and custody of goods rightfully rejected . . . .” UCC 2–715. A non–breaching seller can recover incidental damages for charges incurred in “stopping delivery, in the transportation, care and custody of goods after the buyer's breach, [or] in connection with return or resale of the goods” etc. UCC 2–710.

“Consequential damages” are additional losses incurred by the plaintiff as a result of the defendant’s breach, that usually arise in the area of lost profits following the defendant’s failure to perform on time. To recover consequential damages, the plaintiff must show 1) causation (the damages were a result of the defendant’s breach), 2) the damages were foreseeable at the time the parties entered into the contract, 3) a reasonable certainty as to the amount of damages, and 4) that the damages could not have been mitigated.

©2015 Pieper Bar Review 8

“Costs that were or could have been avoided or mitigated” are simply 1) costs that the plaintiff will no longer have to incur following the defendant’s breach (sometimes referred to as “costs avoided”), like any amounts that the plaintiff no longer has to pay under the breached contract, and 2) losses that the plaintiff can mitigate “by making substitute arrangements for the use of his [or her] resources that are no longer needed to perform the contract” (sometimes called “losses avoided”) Restatement 2d Contracts § 347, comment d.

To avoid pitfalls, always remember that contract law prohibits the recovery of CAPS:

C – CONSEQUENTIAL DAMAGES, unless they were within the contemplation of both parties (foreseeable) when the contract was executed A – Damages that party could have AVOIDED P – Damages for PAIN & suffering or emotional distress resulting from a breached contract, even if such damages were foreseeable S – SPECULATIVE damages (all damages must be proven within a reasonable certainty.

The Caveat: The formula and its component principles are a guide. Any damages formula will work well in some situations, but not work well in others. Our job is to think about concepts like placing the non–breaching party in as good a position as performance would have put her in, and not awarding damages for costs that could have been avoided, and then apply them as rationally as possible. By working through the scenarios to follow, you should gain an understanding of how the courts award damages in a range of different situations, so that you’ll be able to answer any damages questions on the bar exam.

Alternate Method of Looking at Damages: Scholars have developed an alternate measure of damages (not yet embraced by the courts but tested on the Multistate Bar Exam, see e.g. OPE 3 Q98) which breaks down the plaintiff’s recovery into expectation, restitution, and reliance interests. See Joseph M. Perillo, Calamari and Perillo on Contracts 490 (6th ed. 2009).

Expectation Interest = the plaintiff’s expected gain under the contract (essentially lost profits) Restitution Interest = benefits conferred on the defendant that the plaintiff is entitled to recover Reliance Interest = the economic detriment incurred by the plaintiff as a result of the breached contract, which typically includes the restitution interest.

This alternate, modern method embraces the same goal of placing the non–breaching party in as good a position as if the breaching party had performed its obligations under the contract, but uses these interests to categorize the elements of the plaintiff’s recovery. These terms are based on the traditional measures of damages (reliance and restitution).

©2015 Pieper Bar Review 9

Examples:

PROBLEM #1: Breach by Paying Party

Construction Contract Where Homeowner Breaches

1(a). A builder agreed with a homeowner to build a for $100,000, which would have generated a $10,000 profit for the builder. If the homeowner repudiated the contract before the builder began, then the builder could sue for his lost bargain of $10,000, which is the value of the performance (here, the contract price of $100,000), minus what the builder received (here, nothing, since the fact pattern gives no indication that the homeowner was paid anything), minus the cost of completion (here, the $90,000 expenditure avoided by the builder not having to complete performance).

Under the alternate, modern method, the damages would be the same, but the $10,000 would categorized as the plaintiff’s expectancy interest.

1(b). If the builder had partially performed the contract, spending $60,000 on construction prior to the homeowner’s repudiation (anticipatory breach), the builder’s damages would be $70,000, i.e., the value of the performance (here, the contract price of $100,000), minus what the builder received (here, nothing, since the fact pattern gives no indication that the homeowner was paid anything), minus the cost of completion (here, the remaining $30,000 expenditure avoided by the builder not having to complete performance).

Under the alternate, modern approach, the amount needed to make the builder whole would still be $70,000, consisting of a $60,000 reliance interest, plus a $10,000 expectancy interest.

1(c). If the builder had completed the contract, spending $90,000, then he would be entitled to the entire contract price. That could be measured by the value of the performance (here, the contract price of $100,000), minus what the builder received (here, nothing, since the fact pattern gives no indication that the homeowner was paid anything), minus the cost of completion (here, nothing, since the house was completed).

Under the alternate, modern approach, the amount needed to make the builder whole would still be $100,000, consisting of a $90,000 reliance interest, plus a $10,000 expectancy interest.

©2015 Pieper Bar Review 10

PROBLEM #2: Construction Contract Where Builder Breaches

Owner Recovers the Cost of Completion

2(a). A builder agreed with a homeowner to build a basic frame house for $100,000. The homeowner paid the builder upfront, $100,000. After doing most of the job, the builder repudiated (breached) the contract. Other builders would have charged the homeowner $5,000 to finish the job, but the homeowner found builder X, who needed work and was willing to do it for $4,000. How much can the homeowner recover from the breaching builder? Only $4,000, which is the difference between what it cost the homeowner to complete the home ($104,000, including the $100,000 initially paid to the breaching builder and the $4,000 paid to builder X) and the $100,000 contract price. That $4,000 represents the homeowner’s out of pocket cost to remedy the initial builder’s deficient performance. The builder repaying the homeowner $4,000 will put the homeowner in as good a position as performance by the builder would have. That is, a house built for a cost of $100,000 to the homeowner.

Applying the general formula to this rule, the homeowner’s damages are the value of the promised performance, $100,000, minus the value of what was received ($100,000 – remember, that the homeowner in the end received the house he contracted for) (at this point under the formula we are at $0), plus $4,000 incidental damages paid to builder X as a result of the initial builder’s breach.

Under the alternate, modern approach, the amount needed to make the homeowner whole would still be $4,000, consisting of a $4,000 reliance interest. As a result of the breached contract, the homeowner suffered an unanticipated $4,000 cost.

2(b). If instead of paying the builder $100,000 upfront (as described above), the homeowner had initially paid the builder $60,000, the homeowner would owe the builder the remaining $40,000 less the $4,000 cost of completion (i.e. $40,000 minus the $4,000 cost of completion = $36,000). The court would reason that the builder “substantially performed” his obligations (thus making the breach “immaterial”), requiring the homeowner to fulfill her obligation, less the cost of completion.

PROBLEM #3: Repudiation Prior to Performance

Repudiation by Painter on Discounted Performance

A painter contracted with a homeowner to paint the homeowner’s house. The value of such a paint job was $1,200, but the painter needed work and agreed to do the job for $900. Before the homeowner paid any money to the painter, the painter then repudiated the contract.

3(a). If the homeowner is then forced to hire another painter to complete the job for $1,100 (a price reflecting a $100 discount for what we’ve been told was the value of performance), the homeowner cannot recover $900 from the repudiating painter, but only the loss incurred as a result of entering a substitute contract.

That is, the homeowner contracted to receive a painted house for $900. As a result of the breach, she received the painted house for $1,100. Since she is entitled to be placed in as good a position as if the first painter performed (a painted house for $900), the court will award her damages of $200 to reflect the difference between what she paid as a result of the breach ($1,100) and the original contract price ($900).

©2015 Pieper Bar Review 11

3(b). If after the painter’s repudiation, the homeowner decided not to paint her house, she is still entitled to damages her “lost bargain.” That is, contract law allows her to recover as if she had decided to hire another painter. The law sets the price of this substitute transaction at the market price (which the parties can establish through expert testimony). So in this situation, the homeowner would be entitled to recover the difference between the market value of the job ($1,200) and what the homeowner would have had to pay the painter under the contract ($900), resulting in an expectation damages claim of $300 for the homeowner’s lost bargain. Another way of looking at the same problem would be to award the homeowner the value of the performance ($1,200), less the value of what was received ($0), less the cost of paying the painter $900 (a cost avoided), for a total of $300.

3(c). If the painter painted the homeowner’s house defectively (the fact pattern would have to tell you the performance was worth, for example, only $700), then the homeowner’s damages in the event she did not hire someone to fix the paint job, are similar to the scenario above, $500 (the difference between the value of the properly performed paint job ($1,200) and the value of what the homeowner received (a $700 paint job)).

PROBLEM #4: Construction Contract Where Homeowner Breaches

No Recovery for costs that could have been avoided

A builder agreed to build a house for $100,000. When the builder had completed $80,000 of the project, the homeowner repudiated the contract and said she would not pay the builder. The builder then completed construction of the home, incurring an additional $5,000 cost. What can the builder recover from the homeowner?

Consider that the contract price was $100,000, the builder’s reliance interest was $85,000, and his expectation interest was $15,000, but we don’t award damages for “CAPS” (the mnemonic above).

A) $100,000 B) $ 95,000 C) $ 80,000 D) $ 85,000

It cost the builder $85,000 to construct a $100,000 house. From these figures, a court can determine that the builder would have made a profit of $15,000 (his expectancy interest). However, the builder cannot recover the $5,000 expended after the homeowner repudiated the contract, because under contract law a party who stubbornly continues to perform after the other party has repudiated cannot recover losses that could have been avoided ($5,000).

Thus, the answer is B, $95,000, which is the value of what was promised ($100,000), less what was received ($0), plus any consequential or incidental damages ($0), less the $5,000 cost that could have been avoided by stopping work after the repudiation.

Under the alternate, modern approach, the amount needed to make the homeowner whole would still be $95,000, consisting of an $80,000 reliance interest for costs incurred prior to the repudiation, plus the builder’s $15,000 lost profit expectancy interest. As with the explanation above, the builder cannot recover costs that could have been avoided.

©2015 Pieper Bar Review 12

PROBLEM #5: Emotional Distress Arising Out of a Breached Contract A builder agreed to construct a house for a homeowner. The builder knew when the contract was made that the homeowner was in delicate health and the new house was of great importance to her. When the house was done, the homeowner inspected it while the builder waited outside. When the homeowner came out, she slammed the front door and the whole house collapsed. Can the homeowner recover for her emotional distress because of the builder’s breach of contract?

No. Such damages ordinarily are not allowed in a breach of contract action.

Keep in mind that there are rare exceptions to this rule, where the court determines that a severe emotional disturbance was not only foreseeable, but a likely result of a breach. The limited circumstances in which a court has awarded damages for a severe emotional disturbance arising out of a breached contract include breached contracts for the burial of a family member, where a messenger is aware of the contents of a death notification and fails to make a timely delivery resulting in a relative missing a funeral, where a person agrees to be filmed for television on condition that her face is blacked out but his face is shown, etc. John Edward Murray, Jr., Murray on Contracts, § 124 (5th ed. 2011).

PROBLEM #6: Waste

Damages for Immaterial Breach = Cost of Completion (Unless Waste)

When there has been a substantial performance (“HAIL”) in good faith, but a defect exists (especially one which is only incidental to the main purpose of the K), the a court will usually award the cost of completion to the non–breaching party (see Problem #2 above). However, if the cost to correct this minor deficiency is drastically large in proportion to the overall contract price, such that completion to the exact terms of the contract would constitute “economic waste,” the court will award an alternate measure of damages to protect the immaterially breaching party. This alternate measure of damages is the difference between the value of the property as constructed and the value of the property if performance had been properly completed.

For example: A homeowner hired a contractor to build a home for $100,000 and the specifications required, among other things, that the pipes be “galvanized, lap welded pipe of the grade known as 'standard pipe' of Reading manufacture.” The contractor completed the house perfectly, other than that he mistakenly used nearly identical pipe manufactured, not by the Reading Manufacturing Company, but by the Cohoes Rolling Mill Company. The contractor and homeowner agreed that the contractor had substantially performed the contract and that the breach was immaterial, but the homeowner sought damages in the amount of $40,000 (the cost to complete the contract, which would require ripping–up and then refinishing large portions of the house). Here, the courts would say that replacing the Cohoes pipe with identical Reading pipe (the only difference being a stamp on the exterior of the pipe) would constitute economic waste. Therefore, the courts would award as an alternate measure of damages: the difference in value between the house with Cohoes pipe that was received, and the same house with Reading pipe that was called for in the contract. Jacobs & Young v. Kent, 230 N.Y. 239 (1921); City School Dist. v. McLane Constr. Co., 85 A.D.2d 749 (3d Dep’t 1981); Essay #1, Issue 2, February 2012; Essay #1 July 1998 Exam (damages – cost of completion).

However, courts will not award this alternate measure of damages just because the cost of completion is high. If the defect was not incidental to the contract, and the breaching party did not finish the contract, the cost of completion will be awarded even though the difference in value damages might be lower. For example, the owner of a 26–acre industrial site entered a contract with a demolition contractor to sell the scrap metal from its buildings and equipment for $275,000. The contract also required the demolition

©2015 Pieper Bar Review 13 contractor to re–grade the property to make the property more suitable for resale. The demolition contractor removed the buildings and equipment, but failed to re–grade the property, and the owner sued for breach of contract. The owner sought the ordinary measure of damages, i.e. the cost of completion (the cost to re–grade the property), which was $110,000. The demolition contractor argued that this was waste, offering proof that the plaintiff–owner could sell the un–graded property for only $3,000 less than if the property was re–graded (difference in value damages). The court ruled that the demolition contractor owed the $110,000 cost of completion for the breach, noting “[Defendant–contractor’s] completed performance would not have involved undoing what in good faith was done improperly, but only doing what was promised and left undone.” American Standard, Inc. v. Schectman, 80 A.D.2d 318 (4th Dep’t 1981).

PROBLEM #7: Consequential Damages Must Be Foreseeable

A carpenter entered a $10,000 contract to renovate a homeowner’s bathroom by July 1. Based on this contract, the carpenter entered a separate contract to buy a car for $10,000, to be delivered on July 2. The car contract provided that if payment for the car was not made on July 2, the cost of the car would increase to $12,000. The carpenter renovated the homeowner’s bathroom but was not timely paid. Consequently, the carpenter could not pay for the car on July 2, and was required to pay the additional $2,000 for the car at a later date.

The carpenter cannot recover the additional $2,000 from the homeowner, because the breaching party is only liable for those consequential losses that were foreseeable based on the available information at the time of contracting.

©2015 Pieper Bar Review 14

CONTRACT OFFERS

Effective Acceptance Terminated Revocation Unilateral When communicated If the offer cannot be Passage of stated and Contract to the offeree. accepted by a promise time or reasonable New York permit Offer (exception UCC Sale of time, by a revocation anytime Goods contracts), it is counteroffer, an before performance is accepted when contract offeree’s rejection fully completed. performance is fully of the offer, completed. Under incompetency of Restatement (Second) Restatement (Second) either party, or by Contracts § 45 makes Contracts section 45, an intervening the offer irrevocable acceptance occurs when illegally. once performance has an offeree tenders or been tendered or the begins performance, but offeree begins performance must be performance. completed as a condition to any recovery. Bilateral When communicated When acceptance of the Passage of stated Communicated to the Contract to the offeree. offer is dispatched time or reasonable offeree before Offer (“mailbox rule”), or time, by a acceptance has been acceptance is orally counteroffer, an dispatched. communicated to the offeree’s rejection offeror. of the offer, incompetency of either party, or by an intervening illegally. Options When communicated Acceptance of the option Passage of the Not revocable by (Irrevokable to the offeree. (called exercising the stated date or after offeror, and not Offers) Options generally option) is effective only a reasonable time, revoked by offeror’s must be supported by when the acceptance is destruction of the death, or offeree’s consideration received by the offeror. subject matter or rejection or (exception GOL intervening counteroffer. section 5–1109 & illegality. UCC section 2–205 It can be revoked by where a signed the offeror if writing takes the revocation notice place of reaches offeree before consideration). the option does. Auction The bid price (offer) Auction hammer falls and A higher bid The bid is orally Offers is announced by the auctioneer announces terminated a withdrawn by the (UCC 2–328) bidder to the “SOLD.” pending bid. bidder before the auctioneer. hammer falls. Rejection of Only when it is Implied when offeror Acceptance of offer Offeree’s acceptance the Offer received by the receives rejection. is communicated to is mailed, received or offeror. the offeror before is communicated to the offeror receives the offeror before the the rejection. rejection is received by offeror.

©2015 Pieper Bar Review 15

Pre–Existing Duty Rule Summary

Common Law Contracts: A price change had to be supported by new consideration, otherwise the promise to pay the increased price was unenforceable for lack of consideration. The parties’ original price would

govern.

UCC Sales Contracts §2-209(1): The agreed price increase is enforceable without any new consideration if based on “good faith” reason

for asking for more money to perform the contract. No signed writing is necessary unless the modified total price is $500 or more, which then requires a signed writing.

Restatement (Second) Contracts §89: The agreed price increase is enforceable without any new consideration if based on a good faith reason that was not anticipated when the parties originally entered the contract. No signed writing is required to enforce the modification.

New York General Obligations Law §5-1103: In contracts that do not involve the sale of goods, the General Obligations Law abolishes the Preexisting Duty Rule, provided the price modification is contained in a writing signed by the party to be charged with breach of the modified

contract.

©2015 Pieper Bar Review 16

In-Class Multistate Questions: Contracts Breakdown of Contracts on the MBE

50% 50 % Formation of Defenses to Contracts, enforceability, Performance, Parol Evidence and Breach, and Interpretation, Discharge Remedies, and Third-party Rights

Formation, Performance, Breach, and Discharge (50%) What Concepts Do They Expect You to Know?

I. Formation of Contracts II. Performance, Breach, and Discharge

A. Mutual assent A. Conditions 1. 1. Express 2. Indefiniteness or absence of terms 2. Constructive 3. Implied–in–fact contract 3. Obligations of good faith and fair dealing 4. “Pre–contract” obligations based on in performance and enforcement of reliance contracts B. Consideration 4. Suspension or excuse of conditions by 1. Bargain and exchange and substitutes waiver, election, or estoppel for bargain: “moral obligation,” 5. Prospective inability to perform: effect reliance, and statutory substitutes on other party 2. Modification of contracts: preexisting B. and duties C. Discharge of contractual duties 3. Compromise and settlement of claims D. Express and implied warranties in sale–of– goods contracts E. Substantial and partial breach and

©2015 Pieper Bar Review 17

Defenses, Parol Evidence and Interpretation, Remedies, and Third-party Rights (50%) What Concepts Do They Expect You to Know?

I. Defenses to Enforceability

A. Incapacity to contract B. Duress C. Undue influence D. Mistake, misunderstanding E. Fraud, misrepresentation, and nondisclosure F. Illegality, unconscionability, and public policy G. Statute of frauds

II. Parol Evidence and Interpretation

III. Remedies

A. Measure of damages for breach; protecting the expectation interest B. Consequential damages: causation, certainty, and foresee ability C. Liquidated damages and penalties D. Avoidable consequences and mitigation of damages E. Rescission and reformation F. Specific performance; injunction against breach; declaratory judgment G. Restitutionary and reliance recoveries H. Remedial rights of breaching parties

IV. Third-party Rights

A. Third–party beneficiaries 1. Intended beneficiaries 2. Incidental beneficiaries 3. Impairment or of third– party rights 4. Enforcement by the promisee B. Assignment of rights and delegation of duties

© 2015 Pieper Bar Review 18

1. A homeowner advertised his home for 1. Although the painting of the house was sale at an asking price of $100,000. A part of the contract for sale which should prospective buyer expressed an interest in have been in writing, courts allow buying it at the asking price if the recovery on a quasi–contract theory when homeowner would paint the exterior. one party to an oral agreement has The homeowner agreed, but told the performed or partly performed in reliance buyer that his attorney was out of town on the contract. Here, the lead in to the for two weeks and couldn’t draw up the question tells us that the homeowner papers immediately. The parties agreed incurred “costs,” and the reason he that the homeowner would contract to incurred those costs was the oral have the house painted in the meantime. agreement with the buyer. Thus, (A) is an incorrect answer. The next day, the buyer met the painting contractor and selected an unusual shade While the homeowner cannot compel of purple paint for the house. Ten days specific performance of the entire later, when the painting contractor was agreement, he can recover the amount he nearly finished painting the house, the actually expended. Thus, the answer is buyer received a termination notice from (D), not (C). his employer. The buyer immediately informed the homeowner that he would The basis of recovery is not unjust no longer be able to buy the house. enrichment, so (B) is incorrect.

The homeowner approached several realtors regarding the listing of his home, but the realtors informed him that he would need to have the exterior repainted, since the color the buyer had chosen was highly unattractive to most buyers.

If the homeowner sues the buyer to recover his costs in painting the house, is the homeowner likely to prevail?

(A) No, since the homeowner was not bound under an enforceable contract to have the house painted. (B) No, since the painting did not constitute an unjust enrichment to the buyer. (C) Yes, since the buyer breached the contract for the sale of the house. (D) Yes, under a quasi–contract theory, since the homeowner relied to his detriment on the buyer’s promise to purchase the house.

© 2015 Pieper Bar Review 19

2. Which of the following scenarios would 2. A quasi–contract, as distinguished from warrant a quasi–contractual recovery? an implied–in–fact contract, occurs when there is no basis for concluding (A) A fierce blizzard surprised a cross that the parties by their behavior country skier as he skied through impliedly entered into a contract. In the forest. To avoid freezing to (A) and (C), the skier and neighbor death, the skier broke into a nearby impliedly agreed by their conduct to chalet, burned the owner’s pay for the services rendered to him. firewood, and left after the storm Thus, each is liable under the theory of subsided. The owner sued the an implied–in–fact contract, rather than skier for the fair rental value of the quasi–contract. chalet and for the fair market value of the firewood. In (B), the passenger has not by his (B) A plane crash rendered a passenger conduct impliedly agreed to pay for the unconscious for 24 hours. A services rendered. Since he was physician at the scene of the crash unconscious, he could not have rendered medical assistance to the knowingly agreed to or acquiesced to unconscious passenger and later the services provided by the physician. sued the passenger for the value of Therefore (B) is the correct answer. his medical services. (C) A homeowner hired a landscaper to The situation described in (D) also maintain his lawn at 12 Oak Park gives rise to quasi–contract (i.e, where Drive. The landscaper parked his an express contract fails because of the truck, which has an elaborate sign statute of frauds, incapacity of one of identifying his lawn service the parties, illegality, mutual mistake, company, in front of the etc., then in order to avoid the unjust homeowner’s neighbor’s house at enrichment of one of the parties, a 14 Oak Park Drive, and mistakenly quasi–contract may be imposed). The mowed the neighbor’s lawn while measure of damages in (D), however, is the neighbor watched from her incorrect; it is not the value of the window. The landscaper sued the property, but rather the fair market neighbor for the fair value of the value of the niece’s services which will lawn mowing services. be awarded by the court. (D) An elderly uncle told his niece that he would give his house to her if she would live with him and take care of him for one year. At the conclusion of the year, the uncle refused to give his house to the niece, and the niece, unable to obtain specific performance because of the statute of frauds, sued her uncle for the fair market value of the property.

© 2015 Pieper Bar Review 20

3. A student had just completed his junior year 3. An offer for a unilateral contract becomes of college and was looking for a summer job. irrevocable when the offeree tenders He was approached by a homeowner, who performance or commences the work said to him, “I have twenty gallons of paint in requested, but not when there is only mere my garage. If you will paint my house with it preparation to perform that work. The during the next three weeks, I will pay you purchase of paint brushes is only preparation $1,000.” The student immediately went to and therefore the homeowner had the right to the hardware store and purchased three paint revoke the offer at the time he did. (D) is the brushes for $20. That night, the homeowner correct answer. telephoned the student and said, “I’m sorry, but my nephew arrived yesterday, and he (C) is an overly broad statement of the needs the work badly, so I won’t need you to homeowner’s right to revoke. He could not paint my house.” revoke at any time prior to completion, but only before work on the requested project If the student sues the homeowner for breach commenced. Choice (C) would be the of contract, is the student likely to prevail? correct choice under New York law. Peterson v. Pattberg, 248 NY 86 (1928). The (A) Yes, because once an offeree performing party has a cause of action for commences work pursuant to a restitution in quasi contract, but no claim for unilateral contract offer, the offer breach of contract. becomes irrevocable. (B) Yes, because the homeowner induced (B) is incorrect because the reliance in this the student to purchase the paint case, the purchase of $20 worth of paint brushes, and promissory estoppel would brushes, was not of such substantial character effectively convert the homeowner’s that the doctrine of promissory estoppel offer into an . under §87(2) of the Restatement (Second) of (C) No, because an offeror may revoke a Contracts would convert the offer into an unilateral contract at any time before it option contract. The courts will only award a is accepted by completion of the act remedy on the grounds of promissory requested. estoppel where the reliance was substantial (D) No, because the homeowner revoked his (e.g., a substantial expense or forgoing offer before it was accepted. substantial alternatives) and foreseeable. Here the economic injury suffered is not of unconscionable proportion and enforcement of the contract would not be “necessary to avoid injustice.” This answer choice may be tempting since it provides an equitable solution to the student, but it is incorrect (the bar examiners like to provide answer choices that suggest a fair outcome on equitable grounds to throw you off).

(A) is incorrect because the student had not begun the work.

Here, homeowner revoked the offer before the student either tendered performance or commenced the paint job. The student had merely prepared to start the paint job. Calamari, Calamari and Perillo on Contracts § 2.22 at 93–94 (6th ed. 2009).

© 2015 Pieper Bar Review 21

4. A store owner received a written offer 4. A contract can be rescinded for from a clothing wholesaler to purchase unilateral mistake such as this one only 600 Oxford style, white, cotton shirts when the unilateral mistake was so for $2 per shirt, totaling $1,200. The obvious that the other party must have wholesale price for similar shirts is known that the first party made a typically $20 per shirt. The store owner mistake. thought that the wholesaler must have lost his mind, but, nevertheless, agreed This question clearly indicates that a to the offer. The parties each signed a unilateral mistake was made by the purchase order which called for wholesaler, but that fact alone is not delivery of the shirts to the owner’s sufficient reason to permit him to place of business and payment of the rescind the contract. Because (C) is purchase price on July 1. Two days more precise, (A) is not the best later, the wholesaler called the owner answer. and told him that he had made a mistake because he had read from the (B) is clearly incorrect because there wrong line of his price list. He said that was no mutual mistake by both parties. unless the owner paid him $20 per shirt, There is only a unilateral mistake. the price of the shirt on the correct line of the price list, or a total price of (C) is the correct answer. There is a $12,000, he would not deliver the substantial disparity between the shirts. The owner refused to pay contract price of $2 per shirt and the anything more than $1,200. When the market value of the shirts at wholesale, wholesaler failed to deliver the shirts on which seems to be $20 a shirt. If the July 1, the owner purchased 300 buyer, because of the disparity in price, comparable shirts for $20 per shirt and was aware or should have been aware brought suit against the wholesaler for that the seller had made a mistake, the damages resulting from the seller will be able to rescind the wholesaler’s breach of contract. contract and, therefore, will prevail.

Which of the following will the court (D) is incorrect because the owner will conclude? not prevail since he was aware of the mistake. (A) The wholesaler will prevail because he made a unilateral mistake. (B) The wholesaler will prevail because of the doctrine of mutual mistake. (C) The wholesaler will prevail since it was obvious to the owner on May 1 that the wholesaler made a blatant mistake in quoting the price of $2 per shirt. (D) The owner will prevail because he accepted the offer.

© 2015 Pieper Bar Review 22

5. On March 18, a department store published 5. An advertisement indicating a present an ad in the local newspaper that stated: intention to sell to an individual who meets specific criteria (for example, “Now that winter is behind us, we will be being one of the first customers at the selling our inventory of forty mink coats store on the day of the sale) constitutes (which have a retail price of $2,000 per an offer. The owner of the fur store coat) for $600 each, at 8:30 a.m., properly accepted this offer by arriving Saturday, March 20. One coat per person. First Come First Served.” at the store and tendering $600 for the coat. She will not obtain specific The owner of a nearby fur store decided to performance, however, because the coat purchase one of the coats to hold in her was not unique. There were forty in inventory until next fall. She was the first stock at the store, and damages will be customer in line when the department store a sufficient remedy. She will be opened its doors on March 20. She entitled to recover the difference tendered $600 in cash and asked to between the fair market value of the purchase one of the advertised mink coats. coat and $600. Therefore, (B) is correct The manager of the fur department and (A) is incorrect. recognized her as the owner of a competing

fur store and refused to sell her a coat. (C) is incorrect. Although most The owner of the fur store brought an advertisements do not constitute offers, action against the department store seeking the criteria of this particular a judgment that it deliver a coat to her in advertisement were specific enough to exchange for $600, or, in the alternative, constitute an offer. for damages resulting from the department store’s failure to sell her a coat. The advertisement was not to the general public but only to the first forty In her action against the department store, customers who accepted. Customers will the owner of the fur store prevail? reasonably understand that they have

the power to accept when an (A) Yes, on either remedy, because her tender of the $600 was a valid advertisement identifies the quantity to acceptance of the department store’s be sold. Murray on Contracts §35 at offer. 79 (5th ed. 2011). (B) Yes, because the department store breached a valid contract, but the owner of the fur store will only be (D) is incorrect because the “one coat able to collect money damages. per person” limitation does not (C) No, on both remedies, because an reasonably indicate to a purchaser that advertisement to the general public is there is any limitation on who may only an invitation to make an offer, purchase a coat. and when the owner of the fur store made the offer, the department store properly refused to accept it. (D) No, on both remedies, because by placing the “one coat per person” restriction in the ad, the department store was reserving its right to refuse to sell coats to competitors.

© 2015 Pieper Bar Review 23

6. A series of seven arsons occurred at a city’s 6. The January 15 resolution was a unilateral homeless shelter. On January 15, the city contract offer which could be accepted by council adopted a resolution that stated: anyone with knowledge of that offer. It was a specific offer for a specific crime as The city will pay $15,000 for the arrest and contrasted with a standing offer to reward for conviction of anyone found guilty of the any particular crime, e.g., “$10,000 to anyone seven arsons at the city’s homeless shelter. who provides information leading to the capture of any killer of a police officer.” The city council proceedings were telecast This latter type offer can be claimed even if live over the city’s local access cable the offeree was unaware of it. channel. No other publicity was disseminated by the city. Restatement (Second) Contracts § 23 provides that “[s]tanding offers of rewards On January 25, the city council passed a made by governmental bodies [are] intended resolution repealing its reward offer because to create a climate in which people do certain the city was facing budget constraints. The acts in the hope of earning unknown January 25 proceedings were not telecast rewards.” Illustration 3 under § 23 explains, over the local access cable channel because “A city ordinance provides a standing reward the channel was televising the city’s high of [$50,000] will be paid for information school basketball tournament that night. The leading to the arrest and conviction of anyone city council documented the resolution in the guilty of [any] arson within the city limits. A minutes of its proceedings, and the minutes furnishes such information. A is entitled to were published in the legal notices in the the reward whether or not he knew of the back pages of the local paper. reward or was motivated by hope of the reward.” Id. Here, the January 25 resolution On February 1, a bar patron overheard a revoking the earlier offer was not publicized conversation implicating a disturbed in such a way as to revoke the offer except as electrician in the arsons. Since he knew of to those who had actual knowledge of the the reward, but had not heard of its revocation. (A), which incorporates both of revocation, the patron relayed the information these concepts, is the correct answer. to the police. The police lawfully arrested the electrician who confessed and was (B) is incorrect because it suggests that the subsequently convicted of the arsons. The bar patron could have claimed the reward bar patron sought to recover the reward even if he had not known of the January 15 money, but the city refused. reward offer. A person who acts without knowledge of an offer is generally incapable If the bar patron sues the city to recover the of enforcing a contract. The only exception reward, will he prevail? is standing offers of reward made by the government. (A) Yes, because he knew of the January 15 resolution at the time he talked to the (C) is incorrect because the January 25 police and had no knowledge of the council proceeding was not televised. Since January 25 resolution repealing the the purported revocation did not receive city’s offer. publicity comparable to the January 15 offer, (B) Yes, because he had no knowledge of it would be ineffective. Murray on Contracts the January 25 resolution repealing the § 43 at 121 (5th ed. 2011); Calamari, city’s offer and his knowledge of the Calamari and Perillo on Contracts § 220(d) January 15 resolution is irrelevant. at 81–82 (6th ed. 2009). (C) No, because the January 25 resolution effectively revoked the offer. (D) is incorrect because the offer was (D) No, because the offer was not accepted accepted within two weeks from the time it within a reasonable time. was made, which is a reasonable time for a reward offer to remain outstanding.

© 2015 Pieper Bar Review 24

7. Prior to the start of an auction, each 7. Auctions of personal property are bidder and the auctioneer signed governed by UCC §2–328. documents sufficient to satisfy the statute of frauds should a contract be (A) is incorrect because, under UCC formed at the auction, and the §2–328(3), a bidder may retract his bid auctioneer announced that the auction at any time before the auctioneer’s would be “without reserve.” The hammer falls. auctioneer then began to auction a valuable Picasso painting. The first (B) is incorrect because, under UCC bidder bid $100,000 for the painting. A §2–328(3), the retraction of a bid does second bidder immediately thereafter not revive a prior bid. bid $125,000. One minute later, the second bidder withdrew his bid. No If the auction sale is “without reserve,” further bids were made and nothing the auctioneer cannot withdraw the relevant was said during the next five article from auction. (D) is therefore minutes. At that time, the auctioneer incorrect. announced that the auction sale was terminated. However, if there are no bids within a reasonable time, an auctioneer can Which of the following is a correct terminate an auction that is “without statement of the parties’ legal rights and reserve.” Since this occurred, (C) is duties? correct.

(A) There is a binding contract between the auctioneer and the second bidder for $125,000. (B) There is a binding contract between the auctioneer and the first bidder for $100,000, because the second bidder’s withdrawal revived the prior bid. (C) The auctioneer is not contractually bound to sell the painting, because no bids were made within a reasonable time of the second bidder’s revocation. (D) The auctioneer is not contractually bound to sell the painting, because he could withdraw the painting from auction at any time before his hammer fell.

© 2015 Pieper Bar Review 25

8. On June 1, a property seller mailed a signed 8. (A) is incorrect because the June 3 letter letter to a buyer which stated: would best be characterized as an inquiry. An inquiry does not constitute a counter– “I will sell you my house at 23 Garden Lane in offer and therefore is not a rejection of the Louisville for $150,000. I want a deposit of original offer. $15,000 and will close at the registry of on August 1st. Please reply by June 15.” (B) is incorrect because the June 5 letter was a reply to the inquiry of June 3, and did not On June 3, the buyer wrote back: by its terms revoke the June 1 offer. This choice incorrectly characterizes the June 3 “Thank you. Would you consider selling for letter as a counter–offer, when it was really $145,000?” only an inquiry.

On June 5, the seller wrote to the buyer: (C) is incorrect because the additional terms

set forth in the June 7 letter were terms that “I have received your letter of June 3. I reject your offer to buy my house for $145,000.” were implied in the original offer (namely, that the seller convey marketable title). As On June 7, the buyer wrote to the seller: we will cover in further detail in real property, if a contract for the sale of real “I have received your letter of June 5, and property makes no mention of the quality of accept the offer contained in your letter of title to be conveyed, it is implied in the June 1. I am enclosing herewith my check in contract (not in the ) that the seller will the amount of $15,000 as the deposit you tender marketable title at closing. requested. Please deliver a deed to the property conveying marketable title at the (D) therefore is correct because the buyer closing on August 1, at which time I will pay accepted the seller’s June 1 offer though his to you the balance of the purchase price.” June 7 letter.

On June 9, the seller wrote to the buyer:

“I am returning herewith your check for $15,000. I am not contractually obligated to sell you my house at 23 Garden Lane, Louisville.”

The buyer appeared at the Louisville registry of deeds on August 1, prepared to tender the entire $150,000 purchase price, but the seller did not appear. The buyer brought an action against the seller for specific performance of the contract.

Will the buyer likely prevail?:

(A) No, because his June 3 letter was a counter–offer. (B) No, because the seller’s June 5 letter revoked his June 1 offer. (C) No, because the buyer’s June 7 letter was not an effective acceptance because it contained additional terms. (D) Yes, the buyer will prevail.

© 2015 Pieper Bar Review 26

9. A university advertised for bids for a new ice (C) Yes, because a written offer must be hockey arena. The bids were to be submitted revoked by a writing. June 1 and opened on June 15, at which time (D) No, because the dealer validly revoked the contract would be awarded. One of the his offer before the contractor accepted major items included in the bid was a large it. compressor. ~ ~ ~ ~ ~ 9. A similar scenario was presented in A general contractor planned to bid on the Essay #1 on the February 2001 New project and asked several compressor dealers York Bar Exam. to bid on the required compressor. He indicated to the dealers that if he won the contract, he would enter into a contract to buy (B) is correct. The offer made by the the compressor from the lowest bidder on dealer to the contractor is governed by June 20, with delivery to take place on the rules of promissory estoppel set forth August 20. On May 15, a large compressor in §87(2) of the Restatement (Second) of dealer submitted a written offer to sell to the Contracts. When the dealer made the contractor the required compressor for offer, he knew it would induce substantial $75,000. This was the lowest offer received reliance by the contractor in bidding on by the contractor. On May 20, the contractor the skating rink contract. Therefore, an called the dealer, stated his intent to bid on option contract was created and the the university ice hockey arena, and asked if option was properly exercised by the the dealer would hold his offer open until June 21. The dealer said yes. The contractor contractor. then submitted a $5,000,000 bid to construct the ice hockey arena. (A) is wrong because the promise to keep an offer open under UCC §2–205 must be The university awarded the contract to the in writing. Here, the dealer’s promise to contractor, who was the lowest bidder, on keep the offer open was oral. June 15. On June 16, the dealer learned that the contractor received the contract, (C) is wrong because the offer, if it was immediately called the contractor, and revocable at all, could have been revoked revoked the offer to sell the compressor for by a telephone call. $75,000. On June 17, the contractor wrote to the dealer and accepted the offer to sell the compressor for $75,000. When the dealer did (D) is incorrect because the offer was not not deliver the compressor on August 20, the revocable before June 21 for the reasons contractor purchased the compressor set forth in (B). elsewhere for $90,000. In an action by the contractor against the dealer to recover the $15,000 difference between the purchase price and the amount of the dealer’s offer, will the contractor prevail?

(A) Yes, because the dealer’s offer was a “firm” offer under the and could not be revoked until June 21. (B) Yes, because the dealer’s agreement to keep the offer open until June 21 was enforceable since the dealer knew that the offer would induce substantial reliance on the part of the contractor.

© 2015 Pieper Bar Review 27

10. On March 16, an artist mailed to a 10. A rejection terminates the offeree’s patron an offer to paint the patron’s . A counter–offer portrait for $2,500. On April 1, the (a new proposal), because it is an patron mailed back his acceptance, but implied rejection of the offer, has the conditioned the acceptance upon the same effect as an express rejection. painter framing the portrait in a frame Restatement (Second) Contracts § 39. worth at least $500. Thus, (C) is not correct because the artist’s obligation to obtain the frame On April 2, the patron found in his attic was a counter–offer that the artist never a frame perfect for the portrait. That agreed to. same day, the patron faxed to the artist an acceptance of the artist’s offer and (B) otherwise would be correct except advised the artist that he had a suitable for the fact that a rejection (the April 1 frame in which to put the painting. letter) is effective only when received (April 3) by the offeror (the artist). (B) On April 3, the artist received the is not correct because, before the artist patron’s April 1 correspondence. received the counter–offer (rejection), he received the April 2 acceptance by What is the status of the parties fax. Thus, there was a binding contract relationship on April 4? as of April 2 and (D) is correct.

(A) There is no contract to paint the (A) is incorrect because the mailbox portrait for $2,500 because of the rule states that an acceptance is “mailbox rule.” effective when dispatched and put out of the offeree’s control (dropped into (B) There is no contract to paint the the mailbox), and it applies to an portrait because the patron’s “acceptance” and not a rejection of the counter–offer rescinded the artist’s offer (a counter–offer). A rejection is offer. not effective when dropped into the mailbox, but rather only when received (C) There is a contract to paint the by the offeror, here the artist. portrait for $2,500, and for the artist to supply a $500 frame.

(D) There is a contract to paint the portrait for $2,500 but no obligation for the artist to supply a $500 frame.

© 2015 Pieper Bar Review 28

11. The owner of a chain of fast food 11. The suggestion to recycle fat was not part restaurants entered into a written contract of any bargain between the manager and with a manager to operate one of her the owner, and neither past nor “moral” restaurants for two years at a salary of consideration is sufficient to support the $52,000 per year, payable at the rate of owner’s promise to pay the manager $1,000 per week. After the manager had more money for the job he was already worked at the restaurant for 16 months, contractually obligated to perform for he suggested to the owner a way of $1,000 per week. Since this contract is recycling the fat used to cook french fries governed by common law rules, new or which would result in significant savings additional consideration is necessary to in both purchase and disposal costs for support a modification. Therefore, (C) is the entire restaurant chain. The correct. See Pieper NYAA p. 247 (2014). suggestion was implemented immediately and was very successful. (A) is incorrect because the conferring of a material benefit, if not bargained for, Thereafter, the owner told him that she does not support a contract. The benefit was so pleased with his fat recycling conferred by the manager was not suggestion, that she was raising his pay to “bargained for” as part of the owner’s $2,500 per week for the remainder of the promise since it already had been contract. performed by the manager.

When the manager’s next paycheck (B) is incorrect because the manager is arrived, it was only for $1,000. He already contractually bound to manage approached the owner and she told him the restaurant for the remainder of the that, after talking with her accountant, term, and his promise to live up to his she had decided not to raise his pay, but existing contract cannot furnish would continue to honor the conditions of consideration for a modification of that the original contract. contract.

If the manager brings an action against (D) is incorrect because, at the time the the owner to secure the additional $1,500 owner promised to increase the per week, will he prevail? manager’s salary, the contract had less than one year to run and therefore the (A) Yes, because he conferred a material promise would not be unenforceable benefit on the owner. because of the statute of frauds. (B) Yes, because his promise to manage the restaurant for the remainder of the contract term would make the owner’s promise enforceable. (C) No, because past consideration would not support the owner’s promise. (D) No, because a contract for more than a year in duration must be in writing to be enforceable.

© 2015 Pieper Bar Review 29

12. A talented sailor was tied for the lead in a 12. The carpenter was under a legal duty to prestigious weekly race series, with one week complete the repairs by Saturday. Since the remaining before the final race of the season. facts state that the action was brought in a Unfortunately, his boat began to leak and common law jurisdiction (and not in a required emergency repairs if the sailor jurisdiction recognizing the Restatement expected to compete in the final race. A local (Second) of Contracts) and the contract was carpenter told the sailor that because his not a contract for the sale of goods governed regular boss did not have any work for him by the Uniform Commercial Code, for the coming week, he would repair the consideration is required for a modification boat for $1,200 paid in advance. The sailor of the contract. The carpenter is doing paid the $1,200 on the condition that the exactly what he originally promised to do, repairs be completed so that the boat could and, therefore, there is no consideration to sail the following weekend, and the carpenter support the promise to pay the additional began the repairs. $300. Therefore, (C) is correct.

On Thursday morning the carpenter informed (D) is an attractive choice but wrong, because the sailor that his regular boss needed him for the conduct of the carpenter is not sufficiently an emergency job on Thursday and Friday. unconscionable. He originally agreed to do His boss had offered to pay him $500 per day the work at a low price because he was out of to work for him, so the carpenter told the work, and he started the job in good faith. It sailor he would not finish the boat by was only because his regular boss called him Saturday unless the sailor paid him an and offered him more money that he asked additional $300 on Saturday and agreed not for additional compensation for the sailor’s to sue the carpenter for his anticipatory job. The preexisting duty rule clearly applies breach of contract. Because the sailor was here, making (C) the better answer. anxious to win the race series, he agreed to the carpenter’s proposal. The carpenter (A) is incorrect because of the preexisting finished the boat by Saturday morning. duty rule. The carpenter’s promise is to do When the sailor took possession of the boat, exactly what he is already legally obligated to he refused to pay the carpenter the additional do. $300 promised on Thursday. (B) is incorrect because the sailor is not If the carpenter brings an action against the bargaining with the carpenter to give up his sailor in a common law jurisdiction, will he lawsuit in exchange for agreeing to pay the be able to recover the $300? carpenter an additional $300. Both of these promises are to the sailor’s detriment, and (A) Yes, because the emergency excused the there is no new promise to the carpenter’s carpenter’s original performance detriment deadline, and his promise to complete the job despite the emergency was valid consideration for the second agreement. (B) Yes, because the sailor’s promise not to sue for the anticipatory breach of contract was a bargained–for exchange for the carpenter’s promise to complete the job on time. (C) No, because of the preexisting duty rule. (D) No, because the carpenter’s conduct was unconscionable.

© 2015 Pieper Bar Review 30

13. A man applied for and obtained a 13. (C) is the correct answer. The language $100,000 life insurance policy. The quoted from the policy is a condition policy contained the following precedent to performance, which must language: be satisfied before there can be a recovery under the contract. American “This policy shall not insure against a Home Assur. Co. v. International Ins. death which occurs when the insured Co., 90 N.Y.2d 433, 442–443 (1997); is scuba diving, unless he is diving Security Mut. Ins. Co. v. Acker– with a certified P.A.D.I. instructor.” Fitzsimons Corp., 31 N.Y.2d 436 (1972). Two months after the policy was issued, the man was scuba diving in the (B) is incorrect because scuba diving is company of the senior dive instructor. not excluded from the policy. Only The instructor was certified by S.S.I., a diving without a certified P.A.D.I. training organization similar to instructor will result in application of P.A.D.I., and had 30 years of scuba the exclusion. diving experience. While the man and the instructor were diving, a great white (A) and (D) are also incorrect. Whether shark appeared and attacked the man, the statements are true or whether the killing him instantly. Prior to this death would have resulted under attack, no great white sharks had been different conditions is irrelevant. The spotted in or near those waters for more specific language of the condition is than one hundred years. controlling.

The beneficiary of the policy bought an Thus, a condition precedent to the action to collect on the policy, but the performance of this contract (payment insurance company refused to pay. of money) was that any drowning death by scuba diving be in the presence of What is the insurance company’s best P.A.D.I. instructor. Perillo, Calamari defense? & Perillo on Contracts §11.5 at 362 (6th ed. 2009). (A) The death would not have occurred if the insured had the benefit of a certified P.A.D.I. instructor’s experience during the fatal dive. (B) Scuba diving is a hazardous activity properly excluded from the policy. (C) The insured failed to comply with a contractual condition. (D) The purpose of the policy exclusion was to protect the company against the type of scuba diving fatality which occurred.

© 2015 Pieper Bar Review 31

14. A homeowner and a painter entered into 14. The homeowner has a valid cause of a contract to paint the homeowner’s action against the painter because the house for $700 prior to a wedding that painter was a party to the original was being held in the homeowner’s contract and was not discharged from backyard. The contract specifically his obligations when he delegated the stated that payment was contingent work. upon completion by the wedding date. On the other hand, the handyman, who The next day, the painter was offered a is really a subcontractor, is not liable to more lucrative job, so the painter told a the homeowner because the two are not handyman that he was the owner of the in privity. Indeed, the painter told the house, and that he needed the house handyman that the painter was the painted by the wedding day for $500. owner of the house, so the handyman The painter paid the handyman, who had to know of the actual began work on the job, but failed to homeowner’s interest in the completion complete the job on time. of the paint job. Moreover, because the painter did not relay the urgency of If the homeowner brings an action painting the house, it is unlikely that the against the painter and the handyman, painter can recover from the handyman from whom can he recover damages? as time was not made “of the essence” in the contract between the painter and (A) The painter only. the handyman. (B) The handyman only. (C) Both the painter and the (A) is therefore correct. handyman. (D) Either the painter or the handyman, at the homeowner’s election, but not both.

© 2015 Pieper Bar Review 32

15. A patient owed her physician $25,000 for 15. Generally, an assignment terminates the professional services. The physician orally assignor’s right to collect the contract assigned this claim to her adult daughter as benefit (here, the account receivable), but a wedding gift. Shortly thereafter, after the assignee, absent a filing of a UCC suffering sudden, severe losses in the stock Article 9 financing statement, must market, the doctor assigned the same claim immediately notify the obligor owing the to her stockbroker in a signed writing, in money (here, the patient) of the partial satisfaction of advances legally assignment. Since neither assignee notified made by the stockbroker. Subsequently, the patient, she satisfied her debt by paying the patient, without knowledge of either the physician. Choice (D) is the correct assignment, paid the physician the $25,000 answer. then due, which the physician promptly lost at a horse track, although she remains Choice (A) is not the correct answer solvent. because, even though the “first–in–time” rule generally prevails, it does not apply Assuming that Article 9 of the Uniform when the first assignee was gratuitous. A Commercial Code does NOT apply to gratuitous assignment is revocable until the either of the assignments in this situation, gratuitous assignee gets paid, recovers a which of the following is a correct judgment against the person owing the statement of the parties’ rights and money, or enters a new agreement with the liabilities? obligor. Thus, the physician’s daughter has no claim against either the physician or the (A) As the assignee prior in time, the patient since her rights were revoked by (1) physician’s daughter can recover the reassignment of the same right, as well $25,000 from the patient, who acted at as (2) the payment of the debt to the her peril in paying the physician. physician. Choices (A) and (C) are (B) As the sole assignee for value, the incorrect. stockbroker can recover $25,000 from the patient, who acted at her peril in Although past consideration makes the paying the physician. assignment to the stockbroker non– (C) Neither the physician’s daughter nor gratuitous, because the stockbroker did not the stockbroker can recover from the notify the patient, the patient’s payment of patient, but the physician’s daughter, the debt to the physician (the assignor) though not the stockbroker, can extinguished the patient’s obligation. Thus, recover $25,000 from the physician. choice (B) is incorrect. (D) Neither the daughter nor the stockbroker can recover from (D) is the best choice. Even though patient, but the stockbroker, though the physician’s daughter’s gratuitous not the physician’s daughter, can assignment right was extinguished by the recover $25,000 from the physician. physician’s reassignment of the same right to the stockbroker, and the stockbroker’s right against the patient was extinguished by his failure to give notice to the patient prior to her making payment in full to the physician, none of this extinguished the original $25,000 claim the stockbroker had against the physician which was supported by consideration. Thus, the stockbroker has a claim against the physician.

© 2015 Pieper Bar Review 33

16. A farmer entered into a contract to sell 16. Choice (C) is the correct answer. An her entire tobacco crop to a buyer for assignment for consideration is in effect $100,000, to be paid by the buyer upon a transfer of title to, or ownership of, a delivery. Thereafter, the farmer contract right. The transferee, or purchased a race horse from a breeder, assignee, takes title to this contract right and, in writing, assigned her right to to collect money. In order to revoke the payment from the buyer to the breeder assignment, an action for rescission as payment for the horse. The race would have to be commenced. The horse was diseased when the breeder farmer does have a right to rescind here, delivered it to the farmer and it died the because she had a right to reject the next day. race horse, however, it must be accomplished through court action. If delivery of the diseased race horse constituted a breach of warranty of The farmer cannot revoke the merchantability by the breeder, what is assignment by any other method. the farmer’s right vis–a–vis the Therefore, choices (A) and (B) are assigned rights to the tobacco crop? incorrect.

(A) The assignment to the breeder from Choice (D) is incorrect because the the farmer would be revoked farmer may revoke the assignment. automatically. (B) The farmer could revoke the assignment to the breeder by giving him notice. (C) The farmer could only revoke the assignment through an action for rescission. (D) The farmer could not revoke the assignment by any procedure.

© 2015 Pieper Bar Review 34

17. A real estate investor entered into an 17. In contracts other than UCC contracts, agreement for a builder to construct a time usually is not of the essence unless six bedroom summer home and a three the parties make it an express condition bedroom summer home that the of the contract or the circumstances investor wanted to rent out for the indicate that the parties intended time to summer months. The investor was to be of the essence. pay for each house when each was completed. The builder agreed to have (A) is not correct. “A party [generally] the six bedroom home completed by need not perform on the precise day March 15, and the three bedroom home stated in the contract unless time is completed by March 30. A pipe burst made of the essence.” Calamari, in the three bedroom home during Contracts § 11.18 at 376 (6th ed. 2000). construction and destroyed all of the Note, however, although usually rooms but the den. The six bedroom construction contracts are not made home was not completed until June 15, time of the essence, they may be of the which was too late to take advantage of essence even though it is not expressly the summer rental market. stated. It depends on the parties’ intent, and the circumstances. Id. at n.24. The real estate investor refused to pay Since the circumstances here did not the builder for the six bedroom house indicate that time was of the essence, when the bill was tendered on June 25. and the contract did not specifically provide that time was of the essence, Will the builder succeed in its breach of (B) is the best answer. contract action against the real estate investor? Since the two contracts were divisible, the flood in the three bedroom house (A) No, because the law implies that would extend its completion date, but it the time set forth in the contract is would not affect the completion date for of the essence. the six bedroom house. Thus, (C) is an (B) Yes, because the contract did not incorrect answer. specifically provide that time was of the essence. The UCC does not apply to building (C) Yes, because the flood in the contracts. Thus, (D) is an incorrect three–bedroom home would extend answer. the time for completion of the six– bedroom home. (D) Yes, under the common law, but not if the Uniform Commercial Code was enacted by the jurisdiction.

© 2015 Pieper Bar Review 35

18. The owner of a female horse that won 18. In this case, the veterinarian saw an horse racing’s most prestigious event opportunity to use his unique skills to was unable to breed the horse when its create great value from a retired race racing career ended. The owner entered horse. He was not mistaken concerning into a written contract to sell the horse the ability of the horse to produce for $50,000 to a veterinarian, who offspring. Therefore, there was no truthfully described himself as an avid mutual mistake and (B) is incorrect. horse racing fan. $50,000 was a generous price for a sterile While the veterinarian probably knew thoroughbred, but far less than what the that the owner thought that the horse owner could have obtained if the horse would never have offspring, it was not a had been capable of reproducing. proven fact at the time of the sale that it could not reproduce. The facts do not Using his expertise in breeding animals indicate that it was pregnant at the time that had previously been incapable of of the sale. A mistaken belief about conceiving, the veterinarian extracted possible future events does not form the eggs from the horse and fertilized them basis for rescission for unilateral in vitro. When the owner learned that mistake. This doctrine is reserved for the horse was pregnant, he brought an an obvious mistake of fact known at the action to rescind the sale. time the contract was completed. (D) is therefore incorrect. Will the owner prevail? The owner and the veterinarian (A) No, because he has no basis to bargained at arms length. Although the rescind this valid contract. owner did not disclose his plans, he (B) No, because of mutual mistake. made no misrepresentation. He is not, (C) Yes, because the veterinarian failed in fact, obligated to make full to disclose that he specialized in disclosure of his plans. Therefore, the the breeding of previously infertile owner may not rescind the contract for animals. fraud and (C) is incorrect. (D) Yes, because his ignorance of the fact that the horse was capable of (A) is correct because there is no basis being bred successfully constitutes to rescind this contract. See the two unilateral mistake. cases in the NYAA pp. 594-595 (2014).

© 2015 Pieper Bar Review 36

19. An elderly man whose health was 19. Choice (A) is incorrect because the declining relied on his nephew to facts do not indicate that the nephew manage his finances. The nephew induced the elderly man to enter the would fill out checks for the elderly transaction based on any man and then the elderly man would misrepresentation or that the elderly sign them. man relied on (was deceived by) any misrepresentation. The nephew suggested that the elderly man purchase a boat, and after many Mistake as to value generally is not a discussions, convinced the elderly man basis to rescind a contract. Thus, the to purchase the nephew’s boat for best answer is not choice (D). $20,000, an amount substantially more than the boat was worth. The elderly man cannot successfully argue duress because there were no Does the elderly man have a valid basis economic or other threats brought to to vacate the transaction? bear on him. Recognize that the theory of duress only permits a party to avoid a (A) Yes, based on misrepresentation. transaction where a wrongful act or (B) Yes, based on duress. threat overcomes the free will of that (C) Yes, based on undue influence. party and results in an oppressive or (D) Yes, based on mistake. abusive contract. Here, the elderly man could have easily elected not to purchase the boat. Thus, choice (B) is not the best choice.

Undue influence speaks of “unfair” persuasion by one party misusing a position of trust and confidence, or by using a dominant psychological position in an unfair manner. Unfairness may be established by the excessive or inadequate consideration involved. Choice (C) is the best choice.

© 2015 Pieper Bar Review 37

20. A builder agreed to construct two 20. Since the builder was to receive cottages for an owner. One cottage was separate compensation for building to have three bedrooms and the price each of the two houses, this was a agreed upon was $80,000. The other divisible contract. Even if a party’s was to have two bedrooms and the price performance falls short of that required was $75,000. Upon completing each by the doctrine of substantial cottage, the owner was to pay the performance, a court can avoid builder in full. The owner provided the forfeiture and allow recovery on the plans and specifications, and told the contract by holding that the contract is builder that he was going to rent the divisible (or severable) rather than houses for the summer months. The entire. Under the doctrine of divisible builder agreed to complete the two– contract, a party who is in breach can bedroom cottage by March 1, and the nevertheless recover for part three–bedroom cottage by March 15. performance.

The builder began construction on the Even if the builder does not build the three bedroom cottage on January 5, $80,000 cottage, he has substantially and construction on the two–bedroom performed his obligation to build the cottage on February 1. On February 20, $75,000 cottage and so should be after the three–bedroom cottage was allowed to recover for that cottage three–fourths completed, it was under the contract. Thus, (C) is the destroyed by fire through the fault of correct answer. neither the builder nor the owner. The builder completed the two–bedroom Because the builder can recover for the cottage on February 26. $75,000 cottage regardless of his performance of the other part of the If the builder refuses to rebuild the contract, (A), (B), and (D) are incorrect three–bedroom cottage, will he be able answers. to recover on the contract for the completed two–bedroom cottage? Note, the owner has a claim for damages against the builder for the (A) Yes, because the builder did not builder’s failure to complete the three– have an obligation to rebuild the bedroom cottage. Restatement three–bedroom cottage. (Second) of Contracts §240, illus. 1. (B) Yes, only if the owner waives his claim for the breach of the contract for the three–bedroom cottage. (C) Yes, because the contract is divisible. (D) Yes, because of the doctrine of impossibility of performance.

© 2015 Pieper Bar Review 38

21. A contractor entered into a written contract 21. A promise to pay the debt of another to construct a house for a homeowner in ordinarily must be in writing to be accordance with certain plans and enforceable due to the statute of frauds. specifications for $95,000, payable in However, there is an exception under the installments as the work progressed. No “main purpose rule.” If the main purpose of lien law was applicable in the jurisdiction. the is to further the goals of the guarantor, the statute of frauds is As the house neared completion and the inapplicable. That rule is applicable here, contractor had been paid $90,000, the because the main purpose of the contractor informed the homeowner that his homeowner’s promise was to get his house credit was shaky with plumbing supply finished. Restatement (Second) of company which sold him plumbing Contracts §116 illus. 3 (1981). Therefore, supplies. He explained that he would be (A) is incorrect. unable to obtain delivery of $3,000 worth of sinks and toilets to complete the project, (B) is incorrect because the manager’s unless he paid the company $8,000 which shipment of the goods was valid he owed for the plumbing supplies consideration to support the homeowner’s previously purchased for the homeowner’s promise to pay the debt. house. The promise made by the homeowner was The homeowner thereafter called the to pay the entire $8,000 due for plumbing company on the telephone and told its materials used in the construction of his manager that if he delivered the $3,000 house. Once the main purpose rule takes worth of sinks and toilets which the the promise out of the statute of frauds, that contractor had ordered for the house, the promise is enforceable. Therefore, the homeowner would act as a surety for the homeowner owes the entire $8,000, not just contractor’s outstanding obligation to pay the $3,000 for the fixtures shipped, and the $8,000. (D), not (C), is the correct answer.

The manager agreed and shipped the sinks and toilets. Thereafter, the contractor filed for bankruptcy without having made any payment to the plumbing supply and without completing the house for the homeowner.

If the plumbing supply company brings an action against the homeowner, will the plumbing supply company prevail?

(A) No, because the homeowner’s promise to pay the $8,000 is unenforceable because of the statute of frauds. (B) No, because the contractor was already obligated to pay for the plumbing supplies, and therefore the homeowner’s promise was not supported by consideration. (C) Yes, and it will recover $3,000. (D) Yes, and it will recover $8,000.

© 2015 Pieper Bar Review 39

22. When is the court most likely to invoke 22. “A writing that is final is an integration the ? of the terms embodied in it. When it is final and complete it is a total (A) When it discovers evidence of fraud. integration. A writing that is final, but (B) When parties seek to introduce does not completely express the parties’ evidence concerning a consistent contract is a partial integration.” Perillo, term in a fully integrated contract Calamari & Perillo on Contracts §3.2 at with a merger clause. 107 (6th ed. 2009). (C) When parties seek to introduce evidence concerning a consistent “Thus, a partial integration may not be additional term on a partially contradicted by what has been called integrated contract. ‘parol evidence.’ A total integration not (D) When parties seek to introduce only cannot be contradicted by the type evidence explaining the trade of evidence in question but cannot even of a critical contract term in be supplemental by consistent (non– a fully integrated contract. contradictory) additional terms.” Id.; Restatement (Second) Contracts §210(1) and Comment a.

“A merger clause states that the writing is a final, complete, and exclusive statement of all of the terms agreed on. Williston’s first rule, which is followed by most courts, is that a merger clause will ordinarily resolve the issue of total integration.” Perillo, Calamari & Perillo §3.6 at 122 (6th ed. 2009). Thus choice (B) is the best choice for invoking the parol evidence rule.

The general rule is that parol evidence is admissible to show fraud in the inducement. Thus, choice (A) is not correct.

A partial integration, being final but incomplete, may be supplemented by consistent additional terms.” Perillo, Calamari & Perillo § 3.2 at 107 (6th ed. 2009). Thus, the parol evidence rule would not be a bar to choice (C).

UCC 2–202 provides that even a writing intended by the parties as a final expression of their agreement may be explained or supplemented by a course of dealing, by trade usage or by a course of performance. Thus parol evidence could be used in choice (D).

© 2015 Pieper Bar Review 40

23. A restaurant owner and a contractor 23. (B) is correct because, under the entered into an agreement for doctrine of substantial performance, the construction of a restaurant. The plans contractor can recover on the contract. and specifications required the Although the contractor did not contractor to install an “Airflow completely or exactly conform to the Heating System.” About a week before specifications of the contract, he the heating system was to be installed, substantially performed. Under the the contractor discovered that it would common law rule, substantial take four months to obtain an “Airflow performance is enough to allow a party Heating System,” but that a “Flowheat to recover on a contract, though he may Heating System” was immediately still be liable for damages resulting available. Since, the contractor was from his technical breach. See Essay 3, unable to contact the owner to receive July 2008. his permission for the change, he consulted a heating engineer, who To require the contractor to remove the determined that the “Flowheat” system “Flowheat” system and the cost of was as good as the “Airflow” system, it reinstalling an “Airflow” system would would heat just as well, and that it was result in economic waste. of somewhat better quality. Relying upon this advice, the contractor (A) is not applicable to this kind of installed the Flowheat system. The situation. Generally, the doctrine of restaurant owner objected to the impossibility only applies to situations substitution of the Flowheat system and where the contract has become illegal, refused to pay the contractor. the subject matter of the contract has been destroyed, or a party to a personal In an action by the contractor against services contract has died or become the restaurant owner for damages, disabled. which of the following would be the amount of damages recoverable by the (C) is incorrect because the owner was contractor? not available, so the contractor could not contact him. (A) The contractor can recover because of the doctrine of impossibility of (D) is incorrect because the doctrine of performance. substantial performance allows (B) The contractor can recover under recovery on the contract itself, not in the doctrine of substantial quasi–contract. performance. (C) The contractor cannot recover because he failed to contact the owner. (D) The contractor can recover in quasi–contract.

© 2015 Pieper Bar Review 41

24. A renowned financial planner entered 24. The contract for an estate plan is a into a written contract with a woman to personal service, and so a specific prepare an estate plan for a fee of performance remedy is not available. (A) $10,000, to be paid upon the completion is therefore correct. of the plan. The woman retained the financial planner because of his (B) is incorrect because the financial reputation, skill and experience in planner cannot effectively delegate his financial planning. However, the woman obligations under a contract requiring his and the financial planner got along poorly skill and expertise. and, after the second conference, when only a draft of a plan dealing with life (C) is incorrect. Although it is the reason insurance and real estate had been why the delegation to the associate was submitted to the woman, they had a improper, it does not support an award of profound disagreement about the proper specific performance. way to handle the woman’s stock portfolio. Hoping to avoid such (D) is incorrect because the financial encounters in the future, the financial planner’s completion of a portion of the planner, without the woman’s work does not become the basis of a knowledge, assigned all of his rights and specific performance remedy for the duties under his financial planning remainder. contract with the woman to an associate who had recently graduated from college. The associate expressly promised the financial planner to carry out the work to the best of his ability.

The woman, upon learning of the assignment, refused to allow the associate to proceed with the financial plan and brought an action against the financial planner to compel him to resume and complete performance of the contract.

Is the woman entitled to such relief?

(A) No, because the financial planner has contracted to render personal services to the woman. (B) No, because the financial planner effectively delegated his remaining duties under the contract to the associate. (C) Yes, because the financial planner is a preeminent financial planner and his services are unique. (D) Yes, because the financial planner has personally completed a substantial portion of the project.

© 2015 Pieper Bar Review 42

25. A seller agreed to sell 5,000 widgets to 25. (A) is the correct answer because the a buyer for $15,000. The goods were to buyer’s phone call constituted an be delivered on or before December 15. anticipatory breach of the contract On December 5th, while the goods between the buyer and seller. Since this were in transit but before the goods is a contract for the sale of goods, UCC were delivered, the buyer called the Article 2 applies. Under Article 2, a seller and told him that she did not want seller has the right to stop goods in the goods and would not accept them transit when there has been an because she had purchased them anticipatory breach, but only if they cheaper somewhere else. were sent in carload lots (for example, an entire freight car of widgets). What right did the buyer’s telephone Therefore, (B) is incorrect. call give the seller? (D) is incorrect because a seller has the (A) The right to stop the goods in right to stop the goods in transit upon transit if they were sent in carload the buyer’s anticipatory breach lots. regardless of whether the buyer was (B) The right to stop the goods in insolvent. Furthermore, the facts do not transit even if they were not sent in indicate that the buyer was insolvent, carload lots. but they do indicate that the buyer (C) The right to stop delivery of the anticipatorily breached the contract. If goods even if they had reached the the buyer was insolvent, the seller buyer’s city and the carrier had would have the right to stop the goods acknowledged to the buyer that it in transit whether or not they were sent was holding them for her. in carload lots. (D) The right to stop the goods in transit only if the buyer is (C) is incorrect because a seller loses insolvent. her right to stop goods in transit once a carrier has acknowledged the buyer’s right to the goods.

© 2015 Pieper Bar Review 43

26. A buyer and seller entered into a 26. Ordinarily, the risk of loss passes contract for the purchase and sale of entirely to the buyer when the seller 1,000 widgets at a price of $5 per completes his delivery obligations. widget. The widgets were to be Since the contract specifies F.O.B. shipped F.O.B. the buyer’s plant, and buyer’s plant, the risk ordinarily would were to be delivered to the plant on or not pass until the goods actually reach before December 1. The seller shipped the buyer. However, if the buyer them out on November 27. Later in the repudiates or breaches the contract, the day on November 27, before the buyer risk of loss immediately shifts to the received the widgets, the buyer called buyer as to goods that are identified, to the seller and informed the seller that, the extent that the seller’s insurance since widgets were no longer popular fails to cover the loss. UCC §2–510(1). gift items, he was cancelling the order. Therefore, (C) is the correct answer, The goods were destroyed by fire after since the buyer repudiated the contract. the buyer’s telephone call. Note, that this same F.O.B. risk of loss At the time of the fire, which party bore rule was tested on Essay #1 of the July the risk of loss of the widgets? 2003 New York Bar Exam.

(A) The buyer, because the risk of loss (A) is incorrect. Since the contract shifted as soon as the seller specifies F.O.B. buyer’s plant, the risk delivered the goods to the carrier. of loss did not shift when the seller (B) The buyer, because she repudiated delivered the goods to the carrier. the contract. (C) The buyer, but only to the extent (B) is incorrect because the risk of loss that the seller’s insurance failed to only passed to the buyer to the extent cover the loss. that the seller’s insurance did not cover (D) The seller, because the contract the loss. specified “F.O.B. buyer’s plant.” (D) is incorrect because the risk of loss shifted to the buyer, in part, when he repudiated the contract.

© 2015 Pieper Bar Review 44

27. A manufacturer of off–road construction 27. The dealer was insolvent between vehicles, entered into a written contract February 20 and March 20 under the with a dealer to sell to the dealer three definition of insolvency under § 1–201 of large trucks for $30,000 each. The the UCC because he could not pay his manufacturer was to deliver one truck on debts when they were due. Therefore, the March 1, one on April 1, and one on May manufacturer, after demanding the return 1. Payment for each of the trucks, in the of the truck within 10 days of delivery, amount of $30,000, was to be made could reclaim the truck delivered on within 5 days after each delivery. The March 1st under § 2–702 of the UCC. manufacturer delivered the first truck on Therefore, (A) is an available remedy and March 1. an incorrect answer.

On March 4, the dealer called the (B) is also an available remedy and, manufacturer and informed him that, therefore, a wrong answer. If an because he had encountered a delay in insolvent buyer defaults on a payment obtaining a bank loan, he could not pay obligation under an installment contract, for the truck delivered on March 1 until the seller is not obligated to ship in the March 20. The dealer, although he had future on credit, but can demand payment been a reputable heavy equipment dealer in cash and can also demand that the for 25 years, had not paid for vehicles in insolvent buyer pay for any prior a timely fashion since February 20. The delivered installments before any further dealer still has possession of the truck installments are delivered. UCC § 2– delivered on March 1. 702(1).

Which of the following remedies is NOT (C) is also an available remedy and, available to the manufacturer on March therefore, a wrong answer. Where there 5? is a breach of the obligation to pay for one installment, the seller can sue for that (A) Immediately demanding a return of installment prior to completion of all the truck delivered on March 1, and deliveries, if the amount of each reclaiming that truck. installment is clear, as it is in this case (B) Refusing to deliver the truck ($30,000 per installment). scheduled for April 1 delivery unless the dealer pays for the first truck (D) is the correct answer because the before then and pays for the April 1 seller cannot terminate the entire truck in cash on or before the time of installment contract when the buyer delivery. failed to pay only one installment on (C) Suing the dealer for the $30,000. time, even when the buyer is technically (D) Terminating the contract for the insolvent. “An installment contract is April 1 and May 1 trucks. one which requires or authorizes the delivery of goods in separate lots to be separately accepted....” UCC § 2–612.

The seller may, however, demand assurances of payment before he makes future deliveries and can require payment in cash at the time of delivery.

© 2015 Pieper Bar Review 45

28. A buyer and a seller entered into a 28. The notice given by seller to buyer written contract for the sale of 10,000 amounted to an anticipatory breach of lawnmowers, for delivery on February contract by seller, since it 1. On January 10, the seller notified the unequivocally stated that it would not buyer that due to a fire in its factory, the be able to ship conforming goods in the factory would remain closed until quantity stated at the time required. February 1, but the seller could Such an anticipatory breach gives the guarantee delivery no later than buyer, among other remedies, the right February 15. The buyer rejected the to cover: i.e, the right to purchase the seller’s proposal, immediately goods elsewhere and to sue the seller contacted with another lawnmower for the difference between the price the distributor to obtain 10,000 buyer had to pay for the goods and the lawnmowers at a slightly higher cost, original contract price. The buyer did and brought an action against the seller this and now has a right to sue in to recover the difference between the contract for damages. Thus, (D) is the cost of the replacement lawnmowers correct answer. and the original contract price. The buyer’s suit is based on breach of In the buyer’s action against the seller, contract. The seller has committed no will the buyer prevail? tort against the buyer. Therefore, (C) is incorrect. (A) No, because the seller gave assurances of its willingness and (A) is incorrect because of the perfect ability to perform the contract in tender rule under UCC Article 2. This February. rule does not allow for a unilateral (B) No, because the buyer’s lawsuit is delay in performance to a later, premature before February 1. reasonable time when the seller has (C) Yes, because buyer’s complaint unforeseen difficulties. alleges an actionable tort by seller. (D) Yes, because buyer’s complaint (B) is incorrect because the buyer does alleges an actionable breach of not have to wait until the date of contract by seller. performance when there has been an anticipatory breach.

© 2015 Pieper Bar Review 46

29. On October 1, a buyer entered into a 29. UCC 2–210 states that the assignee of a written contract with a seller for the sale of goods contract impliedly assumes purchase of 1,000 mechanical dogs at $20 the full duties of performance under the per dog, to be specially manufactured by terms of that contract unless it is clear the seller according to the buyer’s from the circumstances of the assignment specifications. In order to obtain that the assignee did not intend to assume operating funds, the seller, as borrower, the delegation of duties. entered into a written agreement on October 5 with a finance company. In The purpose of the assignment of the relevant part, this agreement recited, contract to the finance company was to “Seller hereby transfers and assigns to give the finance company a security the finance company its October 1 interest to insure repayment of the loan. mechanical dog contract with Buyer, It was only an assignment of the benefits as security for a 50–day loan of of the contract, not a delegation of the $15,000, the advance and receipt of duties. The finance company never which are hereby acknowledged by expressly or impliedly agreed to perform Seller ....” No copy of the agreement, or the obligations of the contract and statement relating to it, was filed in an therefore, was not liable for a breach of office of public record. the contract. Therefore, (A) is correct.

By November 16, the seller, without legal (B) is incorrect because the filing of the excuse, had delivered no dogs, but the contract would only be material if there buyer brought no action against the seller. was a question of priority among competing security interests, a problem If the buyer brings an action against the not present here. The critical issue with finance company on account of seller’s respect to the finance company’s liability default, will the buyer prevail? is whether it accepted the obligations of the contract, which it did not. (A) No, because the October 5 assignment by the seller to the (C) is incorrect because there was no finance company was only an intent to benefit the buyer in the security assignment for security. assignment made by the seller and the (B) No, because no record of the finance company, and so the buyer was October 5 transaction between seller not an intended third party beneficiary. and finance company was publicly filed. (D) is incorrect because there was no (C) Yes, because the buyer was a third– –– no agreement by the buyer, party intended beneficiary of the the finance company, and the seller to October 5 transaction between seller substitute the performance of the finance and the finance company. company for that of the sellers on the (D) Yes, because the October 5 contract. transaction between the seller and the finance company affected, with respect to buyer as creditor, a novation of debtors.

© 2015 Pieper Bar Review 47

30. A snowmobile manufacturer contracted 30. (B) is correct. The installation of five with an engine company to purchase engines constitutes acceptance of those 100 engines for $250 each. The engines because it is a use which is expressly required that inconsistent with the seller’s rights. each engine would be able to generate The phrase “Despite the lack of 32 horsepower. The contract also sufficient horsepower” is a crucial clue provided that the engines would be showing that the buyer was aware that delivered in lots of 25 and that payment the goods were non–conforming yet would be made within 24 hours after elected to use five of them with delivery. knowledge of the non–conformity.

The first lot of 25 engines was (A) is incorrect. The buyer used five of delivered on June 1, at which time the the engines with knowledge of a defect snowmobile company forwarded and that constituted acceptance. payment to the engine company in the amount of $6,250. However, when (C) is incorrect. Each engine is a later tested, none of the 25 engines distinct commercial unit separable from delivered, or any of the 100 the rest of the lot. A buyer may accept manufactured, could generate more part of a lot and reject the rest, provided than 15 horsepower. he does not break up a commercial unit. Here, the buyer accepted five units and Despite the lack of sufficient properly rejected twenty. UCC § 2– horsepower, the snowmobile company 601. installed five of the engines in snowmobiles. On June 10, the (D) is incorrect. The buyer has the snowmobile company rejected all of the right to accept part of the goods and engines that had been delivered, and reject the rest; hence, acceptance of five advised that it would reject all future engines did not constitute acceptance of deliveries because of the insufficient the total number specified in the horsepower. contract.

On the above facts, how many engines did the snowmobile company accept?

(A) 0. (B) 5. (C) 25. (D) 100.

© 2015 Pieper Bar Review 48

31. A manufacturer entered a contract to 31. The traditional measure of damages for sell a $20,000 printer to a publisher. a total breach of contract by a seller is Just prior to the delivery date fixed in the difference between the market price the contract, the manufacturer notified of the goods at the time of the breach the publisher that it would be unable to and the parties’ contract price. Under deliver the machine. these facts, there are no breach of contract damages since the market price Six months thereafter, the publisher at the time of the contract’s breach purchased the same printer for $22,000. ($20,000) was the same as the $20,000 The publisher then brought a claim contract price. Thus, neither (A), (B), against the manufacturer for the nor (D) is correct. publisher’s lost profits of $1,500 per month for the six months ($9,000) and The UCC also permits a buyer to the additional $2,000 the publisher had “cover”, i.e., make a good faith to pay to for the printer. purchase or enter a contract to purchase substitute goods without unreasonable At the trial, the manufacturer delay. The buyer may then recover the introduced unrefuted credible evidence difference between the cost of cover that if the publisher had purchased a ($22,000) and the contract price replacement printer when the ($20,000). manufacturer breached the contract, the publisher could have readily purchased Here, however, waiting six months to it for $20,000 and would have lost only cover is an unreasonable delay. The one month of profit. publisher’s “damages are measured as of the time the buyer could have What is the maximum amount the covered.... [D]amages cannot be printer is entitled to recover from the enhanced by the buyer’s remaining manufacturer? idle....” Perillo, Calamari & Perillo on Contracts §14.20 at 515–16 (6th ed. (A) $2,000. 2009). (B) $11,000. (C) $1,500. Thus, the publisher can only recover (D) $3,500. $1,500 in consequential damages, i.e, up to the time the publisher could have covered. The publisher cannot recover for all six months of lost profit. Therefore, (B) would not be correct.

(C) is the correct choice. “[A] buyer who fails to cover may be denied consequential damages that could have been avoided by cover.” White & Summers, Uniform Commercial Code § 7–4 at 304 n.43 (6th ed. 2010).

© 2015 Pieper Bar Review 49

32. A coat manufacturer purchased 100 32. The basic rule is that a buyer always bolts of first quality wool from a wool has the right to inspect goods before producer. The sales contract provided accepting them. This is true even if the that the manufacturer would remit buyer has agreed to pay for the goods payment prior to inspection. The wool prior to inspection. Remember to was shipped, and the manufacturer paid separate the buyer’s payment from the the producer. Upon inspection, the right of inspection. manufacturer discovered that the wool was No. 2 quality. Thereupon, the (A) is incorrect because the mere manufacturer tendered back the wool to agreement to pay before inspection in the producer and demanded return of no way constitutes an acceptance of the his payment. The producer refused, goods. An acceptance, remember, contending there is no difference occurs only when: (1) the buyer between No. 1 quality wool and No. 2 signifies that he or she is accepting; (2) quality wool. the buyer uses the goods in some way inconsistent with the seller’s rights; or Which of the following statements (3) the buyer fails to give timely notice regarding the contract provision of rejection. Therefore, (C) is the concerning pre–inspection payment is correct answer. correct? (B) is an absurd answer that is clearly (A) It constituted acceptance of the incorrect. There is simply no goods. connection between the promise to pay (B) It constituted a waiver of the prior to inspection and the buyer’s buyer’s remedy of private sale in remedy of selling the goods in a private the case of nonconforming goods. sale. (C) It did not impair the buyer’s right to inspect the goods prior to (D) is also incorrect, as the clause does accepting or rejecting. not require acceptance before (D) It required the buyer to “accept” inspection; it only requires payment. the goods without inspecting them, but did not affect the buyer’s right to revoke acceptance.

© 2015 Pieper Bar Review 50

33. A publisher purchased 100 cases of 33. (A) is incorrect because specific white 24 lb. paper from a paper performance is only allowed in sale of company. The publisher paid for the goods cases where the property is paper prior to receipt and the paper unique or in “other proper company sent 20 lb. paper instead of 24 circumstances.” There is nothing lb. The publisher promptly tendered the unusual about these goods and nothing paper back to the company and to indicate that damages would not demanded return of its payment. The afford a full remedy. paper company refused, contending the papers to be identical. (B) is incorrect because this is the measure of damages to be used when Assuming that the 20 lb. paper is the buyer accepts the goods. Here, nonconforming, which of the following however, the buyer rejected them. remedies is available to the publisher? (C) states the correct measure of (A) Specific performance. damages because the publisher already (B) Damages, measured by the paid for the goods and then rejected difference between the value of the them as nonconforming. Thus, the goods delivered and the value of publisher can recover restitution conforming goods. damages for the price paid plus the (C) Damages, measured by the price increased cost of cover. paid plus the difference between the contract price and the cost of (D) is incorrect because remedies are buying substitute goods. never waived by agreeing to pay for the (D) None, since the publisher waived goods. his remedies by agreeing to pay before inspection.

© 2015 Pieper Bar Review 51

34. A shoe wholesaler received a catalogue 34. Choice (A) is incorrect because the from a shoe manufacturer which catalogue clearly contains a description described all of the manufacturer’s of the goods (“of the highest quality” is shoes as “all leather except an express warranty). Where an ornamentation” and “of the highest express written warranty is given, a quality.” clause in the sales contract or the bill of sale purportedly disclaiming that Thereafter, the wholesaler and the warranty must fail. manufacturer entered into an agreement (C) is incorrect because the magic whereby the manufacturer agreed to sell words “merchantability”, “as is,” or 10,000 pairs of shoes to the wholesaler. “with all faults” do not appear in the disclaimer clause, and hence the clause The parties’ contract conspicuously in the contract could not operate as a stated: disclaimer of that warranty.

“There are no warranties expressly The disclaimer clause, however, will or impliedly made by seller in operate to negate any implied connection with this sale beyond the warranties of fitness, as a disclaimer of description of the goods contained in that warranty can be made by the use of the seller’s contract.” general terminology. Therefore (B) is correct and (D) is incorrect. Did the disclaimer in the contract negate any of the seller’s warranties?

(A) Yes, all express warranties. (B) Yes, the implied warranty of fitness for a particular purpose. (C) Yes, the implied warranty of merchantability. (D) No, the disclaimer was ineffective.

© 2015 Pieper Bar Review 52